Health Assessment EXAM 1

Pataasin ang iyong marka sa homework at exams ngayon gamit ang Quizwiz!

A client is diagnosed with a scotoma. What question is appropriate for the nurse to ask to obtain more data about this condition? "Do you see floaters in front of your eyes?" "Are the blind spots constant or intermittent?" "How often do you have redness or tearing?" "Is night blindness a problem for you?"

"Are the blind spots constant or intermittent?"

A nurse inspects the gums and teeth of a middle-aged adult and notices the presence of small brown spots on the chewing surfaces of several of the molar teeth. What question should the nurse ask the client to determine the cause of this finding? "Do you drink a lot of coffee or tea?" "Are you experiencing any tooth pain?" "Do you have trouble chewing your food?" "How many cigarettes do you smoke daily?"

"Are you experiencing any tooth pain?"

A client presents to the health care clinic with reports of inability to concentrate at work and daily frontal headaches for the past two weeks. What additional information should the nurse ask this client? "Are you experiencing sinus pressure and congestion?' "Are you taking high blood pressure medication?" "Do you have a family history of severe headaches?" "Have you ever had an injury to your head or neck?"

"Are you experiencing sinus pressure and congestion?'

Which question asked by the nurse is assessing problems with vertigo? "Do you ever have problems with balance?" "Do you experience buzzing in your ears?" "In what situations is it hard for you to hear?" "Have you ever had drainage from your ears?"

"Do you ever have problems with balance?"

A nurse is assessing a client with epistaxis. The client asks the nurse, "What could be causing this to happen so frequently? This is the third time in a month" How should the nurse respond? Select all that apply. "Do you have a history of bleeding disorders?" "Epistaxis is common with acute rhinitis." "This may occur with sinus infections." "Prolonged inhalation of dry air may contribute to this." "Medications that thin your blood may cause epistaxis."

"Do you have a history of bleeding disorders?" "This may occur with sinus infections." "Prolonged inhalation of dry air may contribute to this." "Medications that thin your blood may cause epistaxis."

The nurse is assessing a client who appears to have a swollen thyroid gland (goiter). Which of the following is a priority question the nurse should ask the client? "Do you have difficulty sleeping at night?" "What type of table salt do you use?" "Do you have any difficulty swallowing?" "Has your weight changed over the last few months?"

"Do you have any difficulty swallowing?"

A nurse is teaching nursing students how to conduct a spiritual assessment. The nurse determines understanding when a student states which of the following? "The questions should be specific to the dominant religion." "General introductory questions should be used to start the spiritual assessment." "The nurse should perform a spiritual assessment only if the client requests one." "A spiritual assessment is needed only when the client is experiencing spiritual distress."

"General introductory questions should be used to start the spiritual assessment."

A nurse notes that the pulse rate of a client is less than 60 beats per minute. Which question is appropriate for the nurse to ask the client in regards to this finding? "How is your stress level today?" "What vitamin supplements are you currently taking?" "Have you been sitting for a long time?" "Are you feeling feverish today?"

"Have you been sitting for a long time?"

A female client asks a nurse why it seems like her headaches are more severe and longer in duration than male friends who also have migraines. What is the best response by the nurse? "There is no difference in how migraines affect males and females." "Hormones affect the severity of migraine headaches." "People experience different symptoms with migraines." "The severity of migraines is usually related to genetics."

"Hormones affect the severity of migraine headaches."

A nursing instructor is teaching students about communication in different cultures. When discussing the meaning of hand gestures and body language in different cultures, the nurse realizes that further instruction is necessary when a student makes which statement? "There are many different elements of body language and hand gestures." "If any hand gesture is used, always clarify if there seems to be a strange reaction on the other's part." "I can make a circle with my thumb and forefinger and people of all cultures know it means OK." "A major hand gesture is one for indicating height."

"I can make a circle with my thumb and forefinger and people of all cultures know it means OK."

A nurse is providing care for a client who practices the Jehovah's Witnesses faith. The nurse would determine spiritual distress if the client made which of the following statements? "Fasting gives me strength." "I believe my faith will provide me the strength to get through this." "If I pass on, please do not conduct an autopsy or assess for organ donation." "I don't know if I should refuse the blood products; the doctor says it is a life-saving treatment."

"I don't know if I should refuse the blood products; the doctor says it is a life-saving treatment."

A nurse is conducting a spiritual assessment on a client. The nurse determines the client is spiritual but not religious when the client makes which of the following statements? "I seek peace and meaning in life through meditation." "I enjoy giving back to others by organizing church activities." "I pray to God every day but do not attend church." "Reading the sacred texts is comforting to me."

"I seek peace and meaning in life through meditation."

The client asks the nurse why the nurse put the tuning fork on the bone behind the ear. Which is the best response by the nurse? "It identifies a problem with the normal pathways for sound to travel to your inner ear." "It can identify if you have an inner ear problem causing disequilibrium." "It determines hearing loss caused by degeneration of nerves in your inner ear." "It can determine if you have a problem with repeated ear infections."

"It identifies a problem with the normal pathways for sound to travel to your inner ear."

A client presents at the ophthalmologist's office after being referred by his family health care provider. The referral was made after a benign growth of the conjunctiva was found growing from the nasal side of the sclera to the limbus in the client's right eye. The client asks the nurse what this growth is. What is the best answer the nurse can give? "It is called hypertrophied conjunctiva." "It is called a pterygium." "It is called hypertrophied sclera." "It is called a pterygota."

"It is called a pterygota."

An adult client tells the nurse that his 80-year-old father is almost completely deaf. After an explanation to the client about risk factors for hearing loss, the nurse determines that the client needs further instruction when the client says "There is a genetic predisposition to hearing loss." "Certain cultural groups have a higher rate of hearing loss." "It is difficult to prevent hearing loss or worsening of hearing." "Chronic otitis media has been associated with hearing loss."

"It is difficult to prevent hearing loss or worsening of hearing."

A client comes to the clinic due to losing a fingernail while doing construction on their home. The client asks the nurse how long it will take for the fingernail to regrow. What is the best response by the nurse? "It will probably take about 12 months to totally replace a fingernail." "It takes about 6 months to totally replace a fingernail." "It will only take about a week for it to fully regrow." "It will grow back in time, but may never be the same."

"It takes about 6 months to totally replace a fingernail."

The mother of a small child with tubes in both eardrums asks the nurse if it is okay if the child travels by airplane. What is the nurse's best response? "He should avoid flying for 6 months after tube placement." "The child must wear ear plugs while flying." "It's safe to fly because the tubes will equalize pressure." "He shouldn't fly with anyone who is immunocompromised."

"It's safe to fly because the tubes will equalize pressure."

The nurse is conducting discharge teaching to the caregiver of an older adult who was hospitalized following a fall at home. Which statement by the caregiver indicates a need for additional teaching by the nurse? "Standing up slowly is important because dizziness can cause falls." "Loss of sensation in the toes is an age-related change." "Certain medications can cause muscle weakness." "Changes in vision such as decreased accommodation happen with aging."

"Loss of sensation in the toes is an age-related change."

A nurse is teaching nursing students about the risks associated with developing head and neck cancers. The nurse determines student understanding when the students make which of the following statements? "Asbestosis does not cause head and neck cancers." "Most head and neck cancers are linked to smoking." "Alcohol plays a very little role in head and neck cancers." "Chewing tobacco does not cause cancer."

"Most head and neck cancers are linked to smoking."

A client calls the clinic and tells the nurse that the doctor told her that she has "otalgia." The client cannot remember what the doctor explained this to be. How would the nurse most appropriately respond? "Otalgia is discoloration of the ear." "Otalgia is pain in the ear." "Otalgia is the beginning of hearing loss." "Otalgia is a disease of the inner ear."

"Otalgia is pain in the ear."

The nursing instructor is teaching a class about how to assess pain in older adults. The teachers tells the students that problems can arise in certain circumstances. The instructor realizes the need for more teaching about pain in the elderly when one of the students replies: "Pain is a natural part of aging." "Patients may fear that uncontrolled pain will affect their independence." "Older clients may worry that reporting pain will lead to costly tests." "Patients are reluctant to report pain because they want to be considered as 'good' clients."

"Pain is a natural part of aging."

A nurse is interviewing a 65-year-old client with a history of atrial fibrillation, type 2 diabetes, obesity, and congestive heart failure. The nurse determines the client is experiencing chronic neuropathic pain when the client makes which of the following statements? "My shoulder has been hurting off and on for the year." "The burning sensation in my feet has gotten worse over the past year." "I have had this aching pain in the right side of my stomach for a few months now." "I have been so depressed since my husband died that I ache all over."

"The burning sensation in my feet has gotten worse over the past year."

A client is having their tonsils removed. The client asks the nurse what function the tonsils serve. Which of the following would be the most accurate response? "The tonsils contain nerves that provoke sneezing." "The tonsils aid in digestion." "The tonsils regulate the airflow to the bronchi." "The tonsils help to guard the body from invasion of organisms."

"The tonsils help to guard the body from invasion of organisms."

A client asks why gloves are being worn during the physical examination. What should the nurse respond to this client? "It's a policy I have to follow." "They help me feel your body parts under your skin better." "They make sure that any microorganisms on my hands do not touch your skin." "Since we don't know what's wrong with you, I wear gloves to make sure I don't get sick."

"They make sure that any microorganisms on my hands do not touch your skin."

A nurse is discussing with a client the client's personal health history. Which of the following would be an appropriate question to ask at this time? "Are both of your parents still living?" "What do you usually eat in a typical day?" "What diseases did you have as a child?" "How do you feel about having to seek health care?"

"What diseases did you have as a child?"

A nurse is teaching nursing students how to perform a cultural assessment. The nurse would include which of the following interview question(s)? Select all that apply. "What does being healthy mean to you?" "How do you view the causes of illness?" "What is most important to you?" "What is your current salary?" "Where were you born?"

"What does being healthy mean to you?" "How do you view the causes of illness?" "What is most important to you?" "Where were you born?"

Which of the following questions is most useful in the assessment of a client's diabetes management? "You check your sugars before each meal, don't you?" "Are you still using your glucometer 4 times a day?" "Are you staying vigilant with your blood sugar monitoring?" "What is your routine for checking your blood sugar these days?"

"What is your routine for checking your blood sugar these days?"

A nurse is collecting data on a client's chief complaint, which is a spell of numbness and tingling on her left side. Which of the following questions would be best for eliciting information related to associated factors? "How bad was the tingling and numbness?" "How long did the spell last?" "Where did the numbness and tingling occur?" "What other symptoms occurred during the spell?"

"What other symptoms occurred during the spell?"

An adolescent wrestler has been diagnosed with herpes simplex virus with weeping lesions on the face, nose, and lips. The client asks the nurse when he can resume competition wrestling. What is the nurse's best response? "This condition is not contagious, so there is no need to stop wrestling." "When the repeated cultures are clear, you may resume wrestling." "You can wrestle after the lesions stop weeping and have crusted over." "As long as the lesions are covered, you can wrestle."

"You can wrestle after the lesions stop weeping and have crusted over."

A 71-year-old woman has been admitted to the hospital for a vaginal hysterectomy, and the nurse is collecting subjective data prior to surgery. Which statement by the nurse could be construed as judgmental? "How often do your adult children typically visit you?" "Your husband's death must have been very difficult for you." "You must quit smoking because it affects others, not only you." "How would you describe your feelings about getting older?"

"You must quit smoking because it affects others, not only you."

What are standard precautions for all pt care?

- Hand hygiene - Gloves - Skin/nail care - Respiratory hygiene/cough etiqutte

Which vision acuity reading indicates blindness? 20/20 20/200 20/40 20/100

20/200

A 72-year-old teacher comes to a skilled nursing facility for rehabilitation after being in the hospital for 6 weeks. She was treated for sepsis and respiratory failure and had to be on a ventilator for 3 weeks. The nurse is completing an initial assessment and evaluating the client's skin condition. On her sacrum there is full-thickness skin loss that is 5 cm in diameter with damage to the subcutaneous tissue. The underlying muscle is not affected. What is the stage of this pressure ulcer? 1 2 3 4

3

A nurse examines a client with complaints of a sore throat and finds that the tonsils are enlarged and touching one another. Using a grading scale of 1+ to 4+, how should the nurse appropriately document the tonsils? 1+ 4+ 2+ 3+

4+

A student nurse is learning to document an initial assessment. What would the instructor tell the student that accurate documentation of this specific assessment best provides? Data on the client's prognosis for recovery Information on the effectiveness of interventions A baseline for comparison with future findings Information on the nurse's cultural competence

A baseline for comparison with future findings

An increased risk of falls is dangerous for any client. What client would be at an increased risk of falls? A client with a hearing loss of 45 dB. A client with acute otitis media. A client with vertigo. A client with damage to the VIIIth cranial nerve.

A client with vertigo.

A nurse is assessing a small child who has lead poisoning. Which characteristic of the gums should the nurse expect this client? Enlarged, reddened Red, bleeding Pink, moist, firm A grey-white line

A grey-white line

The nurse notes a tophus of the ear of an older adult. Which assessment data is consistent with a tophus? A hard nodule composed of uric acid crystals A sac with a membranous lining filled with fluid Scarring of the tympanic membrane Redness and bulging of the eardrum

A hard nodule composed of uric acid crystals

When assisting a client with health promotion, what must the nurse also nurture? A healthy environment Knowledge of the Healthy People 2020 indicators Family communication School/work attendance

A healthy environment

The nurse is teaching an older adult diagnosed with diabetes about the skin. Which of the following should be emphasized? A neuropathic ulcer can develop without feeling it. Skin collagen decreases with age. Wound healing becomes prolonged with age. Hydration alters skin turgor.

A neuropathic ulcer can develop without feeling it.

A client has Darwin tubercle. What is this? A type of skin cancer found on the ear A growth in the ear canal A growth in the bony labyrinth A small painless nodule on the helix

A small painless nodule on the helix

As a nurse is adjusting a client's hospital bed, the nurse accidently pinches a finger between the bed and the wall. Which of the following components is involved in the transduction of the pain the nurse feels? neuronal plasticity K-fibers L-beta fibers A-delta and C fibers

A-delta and C fibers

A client is reporting pain and rates it as 7 on a scale of 1 to 10. When the nurse asks him to describe the pain, he states, "It feels like a knife is stabbing or cutting me." The nurse knows that this type of pain is conducted by which fibers? C fibers A-delta fibers AC fibers P fibers

A-delta fibers

Where is the temporal artery palpated? Above the cheek bone near the scalp line Just left of midline at the base of the neck Between the mandibular joint and the base of the ear Just left or right of the spine at the base of the skull

Above the cheek bone near the scalp line

Upon assessing the skin, the nurse finds pustular lesions on the face. The nurse identifies that these could be what? Acne Psoriasis Varicella Herpes simplex

Acne

A client presents to the health care clinic with reports of a 3-day history of fever, sore throat, and trouble swallowing. The nurse notes that the client is febrile, with a temperature of 101.5°F, tonsils are 2+ and red, and transillumination of the sinuses is normal. Which nursing diagnosis should the nurse confirm based on this data? Self-Care Deficit Hopelessness Acute Pain Ineffective Health Maintenance

Acute Pain

A client seeks medical attention for pain when touching the area of the frontal sinuses. Which should the nurse consider as the reason for this client's symptom? Eye infection Acute otitis media Oropharyngitis Acute bacterial rhinosinusitis

Acute bacterial rhinosinusitis

Upon examination of the head and neck of a client, a nurse notes that the submandibular nodes are tender and enlarged. The nurse should assess the client for further findings related to what condition? Metastatic disease Chronic infection Acute infection Cushing's disease

Acute infection

A pathophysiology instructor is discussing pain and its treatment across cultures. The instructor points out that clients from racial and ethnic minorities often receive less pain medication compared to Caucasians for what specific conditions? Acute pain in the ED Chronic pain from fibromyalgia Broken limbs Head injuries

Acute pain in the ED

A client with scabies visits the health care facility for a follow-up appointment. Which preparation by the nurse is of greatest priority for the physical examination of this client? Warm, comfortable room Quiet area free of disturbance Adequate lighting Firm examination bed or table

Adequate lighting

What do retinal abnormalities include? Age-related macular degeneration Mydriasis Argyll Robertson syndrome Horner's syndrome

Age-related macular degeneration

A client is brought to the emergency department by ambulance after a motor vehicle accident. What would be given the highest priority by the staff triaging the client? Breathing Airway Circulation Disability

Airway

Which factor, if present in a client's lifestyle and health practices assessment, would alert the nurse to the need for performing a more thorough head and neck assessment? Select all that apply. Alcohol abuse Recreational drug use Smokeless tobacco use Multiple sex partners

Alcohol abuse Recreational drug use Smokeless tobacco use Multiple sex partners

The student nurse learns that examining the skin can do all of the following except? Reveal overhydration Allow early identification of neurologic deficits Identify physical abuse Allow early identification of potentially cancerous lesions

Allow early identification of neurologic deficits

The nurse is planning to instruct a group of adolescents on ways to prevent traumatic brain injuries. What should be included in these instructions? Always use seat belts. Wear nonslip shoes in the house. Avoid risky activities such as snowboarding. Use of guns should be supervised by an adult.

Always use seat belts.

Diplopia present with one eye covered can be caused by which of the following problems? Weakness of CN III Weakness of CN IV A lesion of the brainstem An irregularity in the cornea or lens

An irregularity in the cornea or lens

What is the term used to describe a pharmaceutical agent that relieves pain? Antacid Antihistamine Analgesic Antibiotic

Analgesic

During a lecture on pain management, the nursing instructor informs the group of nursing students that the primary treatment measure for pain is which of the following? Analgesics Surgery Relaxation techniques Cutaneous stimulation

Analgesics

While examining a client, the nurse observes that he appears to be nodding his head involuntarily. Which of the following conditions should the nurse additionally assess for, based on this finding? Neurologic disorder Aortic insufficiency Unilateral vision Paget's disease

Aortic insufficiency

A 82 year old female presents with neck pain, decreased strength and sensation of the upper extremities. The nurse identifies that this could be related to what? Arthritic changes of the cervical spine Bacterial thyroiditis Cranial damage Muscle tension

Arthritic changes of the cervical spine

When can the general inspection be started? During the examiner's preparation to meet the client When the client is completely exposed After height and weight have been taken As soon as the examiner first sees the client

As soon as the examiner first sees the client

A nurse assesses a client's pupils for the reaction to light and observes that the pupils are of unequal size. What should the nurse do next in relation to this finding? Report this to the health care provider Ask the client about previous trauma to the eyes Document this finding in the client's record Continue with the examination

Ask the client about previous trauma to the eyes

A nurse assesses a newly admitted 43-year-old client and documents the vital signs as follows: temperature 98° F (36.7° C), pulse 93 beats/min regular rhythm and bounding, blood pressure 145/93 mm Hg, and respiratory rate 16 breaths/min. What is the first action of the nurse? Request a prescription from the health care provider. Ask the client if they are experiencing other symptoms. Request a prescription for an antipyretic. Document findings as normal.

Ask the client if they are experiencing other symptoms.

A new order for an antibiotic is received for a client. The nurse reviews the client's electronic medical record. The record states the client has no known allergies. What action should the nurse take? Administer the medication. Ask the client if they have allergies. Double-check in the admission notes for allergies. Hold the medication.

Ask the client if they have allergies.

A nurse palpates a client's cervical lymph nodes and notes the following findings: cervical lymph nodes .6 inches (1.5 cm) in diameter (enlarged), painful, and mobile. What is the best action of the nurse? Document findings as normal. Notify the health care provider. Ask the client if they have a history of cancer. Ask the client if they have experienced any other signs or symptoms.

Ask the client if they have experienced any other signs or symptoms.

Which action by the nurse is appropriate to provide a clear view of the uvula for observation? Ask the client to say "aaah" Ask the client to stick out the tongue Depress the tongue slightly off center Press firmly on the back of the tongue

Ask the client to say "aaah"

A mother of a small child calls the clinic and asks to schedule an appointment for ear tube removal. The call is transferred to the nurse. What is the nurse's best action? Schedule first available office appointment. Schedule appointment at hospital for tubes to be removed surgically. Ask healthcare provider about prescribing antibiotics before removal. Ask the mother how long the tubes have been in place.

Ask the mother how long the tubes have been in place.

Which technique by the nurse demonstrates proper use of the ophthalmoscope? Uses right eye to examine the client's left eye Moves the scope around so the entire optic disk may be seen Approaches the client directly in front of the pupil Asks the client to fix the gaze upon an object and look straight ahead

Asks the client to fix the gaze upon an object and look straight ahead

A client with a cervical spine injury reports chronic pain. What would be the most appropriate initial nursing intervention for this client? Work with medical team to evaluate possible surgery. Discuss pharmacologic interventions. Educate the client regarding cervical spine pain. Assess the client regarding characteristics of the pain.

Assess the client regarding characteristics of the pain.

A client who underwent abdominal surgery this morning reports feeling weak and dizzy. The nurse also observed a decrease in urine output in the last hour. What action should the nurse take first? Assess the client. Administer IV fluids. Evaluate the outcome. Reevaluate the nursing plan.

Assess the client.

A woman newly immigrated to the United States is admitted to the obstetric unit. While doing a transcultural assessment, how would the nurse individualize questions for this client? Assessing if the client speaks and understands English Realizing that some women are not allowed an education in their home country Requesting a professional translator fluent in the woman's language Directing assessment questions only to the client's partner or family members

Assessing if the client speaks and understands English

A client asks a nurse to look at a raised lesion on the skin that has been present for about 5 years. Which is an "ABCDE" characteristic of malignant melanoma? Asymmetrical shape Borders well demarcated Color is uniform Diameter less than 1/8 of an inch

Asymmetrical shape

A client performs the test for distant visual acuity and scores 20/50. How should the nurse most accurately interpret this finding? Client did not wear his glasses for this test and therefore it is not accurate. When 50 feet from the chart, the client can see better than a person standing at 20 feet. Client can read the 20/50 line correctly and two other letters on the line above. At 20 feet from the chart, the client sees what a person with good vision can see at 50 feet.

At 20 feet from the chart, the client sees what a person with good vision can see at 50 feet.

During the physical examination of a client, a nurse notes that a client's trachea has been pushed toward the right side. The nurse recognizes that the pathophysiologic cause for this finding is related to what disease process? Endocarditis Bronchitis Atelectasis Tuberculosis

Atelectasis

Recommended protective measures to avoid skin cancer include which of the following? Avoiding sun exposure Knowing signs of skin cancer Performing monthly skin self-examinations Seeking biannual examination by a clinician after age 40 years

Avoiding sun exposure

The client comes to the health care provider stating he has a sore throat and believes he needs an antibiotic. This is an example of what type of model for health? Biomedical Complementary Spiritual Alternative

Biomedical

A 55-year-old bookkeeper comes to the office for a routine visit. The nurse notes that on a previous visit for treatment of contact dermatitis, the client's blood pressure was elevated. She does not have prior elevated readings, and her family history is negative for hypertension. The nurse measures her blood pressure in the office today. Which of the following factors can result in a false high reading? Blood pressure cuff is tightly fitted. Client is seated quietly for 10 minutes prior to measurement. Blood pressure is measured on a bare arm. Client's arm is resting, supported by the nurse's arm at the client's mid-chest level.

Blood pressure cuff is tightly fitted.

Universal precautions are primarily designed to protect the health care worker from what? STDs Musculoskeletal injuries Blood-borne pathogens Respiratory diseases

Blood-borne pathogens

An alternate pathway that bypasses the external and middle ear is called what? Bone conduction Sensory conduction Neuro conduction Air conduction

Bone conduction

A client reports a 20 pack per year history of cigarette smoking. To assess this client for cancer, where should the nurse inspect the tongue? Posteriorly near tonsils Both sides At the very tip Frenulum

Both sides

A nurse obtains a pulse rate on an adult client of 56 beats per minute. What is the correct term that the nurse should use to document this finding? Normal Bradycardia Tachycardia Hypocardia

Bradycardia

The nurse is palpating a client's cervical vertebrae. Which vertebra can be easily palpated when the neck is flexed and should help the nurse locate the other vertebrae? C1 C3 C5 C7

C7

When inspecting the mouth, the nurse focuses on lateral and vertical surfaces of the tongue and its base, because these are regions where: Early jaundice can be detected. Lesions from loose dentures are found. Sloughing of papillae begins. Cancers often occur.

Cancers often occur.

The nurse is assessing a client who has been taking antibiotics for an infection for 10 days. The nurse observes whitish curd-like patches in the client's mouth. The nurse should explain to the client that these spots are most likely leukoplakia. Fordyce spots. Candida albicans infection. Koplik spots.

Candida albicans infection.

An adult client comes to the ED with a new onset of pain in his neck and jaw. What system requires emergency assessment? Cardiovascular Integumentary Respiratory Nervous

Cardiovascular

The nurse is palpating a client's neck as part of a physical assessment. Which of the following blood vessels should the nurse be especially careful to avoid bilaterally compressing during the assessment? Internal jugular vein Carotid artery External jugular vein Temporal artery

Carotid artery

A nurse is working with a client from Asia who has just been diagnosed with oropharyngeal cancer. Which culture-related risk factor should the nurse most suspect in this client? Heavy alcohol use Infection with human papillomavirus Chewing betel nuts Smoking cigarettes

Chewing betel nuts

The nurse plans time to sit with a client to help address the client's spiritual needs. Why is this important? Clients feel more comfortable talking with nurses Nurses are easier to talk to than spiritual advisors or chaplains Clients are holistic beings consisting of body, mind, and spirit Nurses have the most time to address a client's spiritual needs

Clients are holistic beings consisting of body, mind, and spirit

The nurse is conducting a physical examination of a client. After completing the examination, the nurse realizes that part of the examination was omitted by mistake. How should the nurse proceed? Complete the forgotten portion of the exam out of sequence. Perform the complete assessment at the end of the shift. Inform the client that the examination was incomplete and start over. Proceed as normal with the assessment data collected from the incomplete exam.

Complete the forgotten portion of the exam out of sequence.

A nurse is examining the eyes of a client who has complained of having a feeling of a foreign body in his eye. The nurse examines the thin, transparent, continuous membrane that lines the inside of the eyelids and covers most of the anterior eye. The nurse recognizes this membrane as which of the following? Retina Sclera Cornea Conjunctiva

Conjunctiva

The nurse is caring for a married female client who defers to her husband to answer all assessment questions. The nurse understands that it is common in some cultures for the male to hold a dominant role in the relationship. What stage of cultural awareness does the nurse display? Unconscious competence Conscious incompetence Unconscious incompetence Conscious competence

Conscious competence

A student nurse is palpating the neck of a client who reports a lump behind the ear. While palpating, the student nurse notes that the lump is immobile. Which action by the student nurse is best in response to this finding? Inform the client that the lump may be malignant. Discuss treatment options with the client. Consult with a clinical instructor. Ask another student nurse to confirm the finding.

Consult with a clinical instructor.

A male client with a history of a back injury 2 months ago has been taking daily doses of narcotic pain medication. He is currently hospitalized with a leg fracture after falling down the stairs. He complains of 10/10 pain in his back and leg after taking pain medication one hour ago. What is the nurse's best action? Consult with the healthcare provider about increasing the dose of medication. Inform the client that the next dose of medication is due in one more hour. Request a psychiatric evaluation for drug seeking behavior. Tell the client to take his own prescription medication.

Consult with the healthcare provider about increasing the dose of medication.

A nurse is inspecting the ears of an Asian client and observes that her earlobes appear soldered, or tightly attached to adjacent skin with no apparent lobe. Which of the following should the nurse do next? Notify the physician of the finding Ask the client whether she has ever experienced an injury involving her ears Continue with the examination Record the finding and plan to follow-up at the client's next visit to note any changes

Continue with the examination

When assessing an adult client experiencing diarrhea, the nurse notes a round "moon" face, a buffalo hump at the nape of the neck, and a velvety discoloration around the neck. What is the possible cause of these signs? Myxedema Cushing's syndrome Scleroderma Bell's palsy

Cushing's syndrome

A nurse is working with a 13-year-old boy who complains that he has begun to sweat a lot more than he used to. He asks the nurse where sweat comes from. The nurse knows that sweat glands are located in which layer of skin? Stratum corneum Stratum lucidum Dermis Epidermis

Dermis

Which layer of the skin contains blood vessels, nerves, sebaceous glands, lymphatic vessels, hair follicles, and sweat glands? Dermis Epidermis Subcutaneous layer Connective layer

Dermis

The nurse is caring for an older adult client with a nasogastric feeding tube ordered by the physician. The nurse notes that the client is not a mouth breather and having no difficulty breathing. While inserting the feeding tube, the nurse encounters difficulty getting the tube through the nares. What should the nurse suspect? Swollen nasal passages Deviated septum Obstructed turbinate Hypertrophied adenoids

Deviated septum

A nurse examines a client's retina during the ophthalmic examination and notices light-colored spots on the retinal background. The nurse should ask the client about a history of what disease process? Anemia Renal insufficiency Diabetes Retinal detachment

Diabetes

The nurse is planning a presentation to a group of high school students about the risk factors for oral cancer. Which of the following should be included in the nurse's plan? Diets low in fruits and vegetables are a possible risk factor for oral cancer. About 40% of all cancers occur in the lips, mouth, and tongue. Most oral cancers are detected in people in their 70s. The incidence of oral cancers is higher in women than in men.

Diets low in fruits and vegetables are a possible risk factor for oral cancer.

The terms "generalized," "exposed surfaces," "upper arm," and "skin folds" are used to describe which major characteristic of skin lesions? Type Color Distribution Arrangement

Distribution

A client just died, and the nurse is preparing the body for the funeral home. The client practiced Judaism and a rabbi was present at the time of death. The nurse is careful to do which of the following to honor the client's religious beliefs concerning death? Do not cross the client's arms. Burn all bandages with the client's blood. Discard all clothes of the client. Place a medal with the client.

Do not cross the client's arms.

A nurse is performing an admission assessment of a client of Asian descent who participates in traditional cultural practices. During the assessment the nurse observes large, round, red marks on the client's back. What action should the nurse take? Document findings. Ask the client's spouse to leave the room. Report findings of abuse to the nursing supervisor. Notify the health care provider of abnormal findings.

Document findings.

A nurse cares for a client of Asian descent and notices that the client sweats very little and produces no body odor. What is an appropriate action by the nurse in regards to this finding? Assess the client for changes in sensation due to vascular problems Monitor the client for additional findings of cystic fibrosis Suggest that the client use antiperspirant products Document the findings in the client's record as normal

Document the findings in the client's record as normal

The nurse is conducting a physical examination of a client who is lying down. Which is the most appropriate for the nurse to assess while the client is in this position? Range of motion of the spine Posterior chest excursion Head and neck range of motion Dorsiflexion of the foot

Dorsiflexion of the foot

A nurse is assessing a client before administration of an antibiotic. The nurse should be aware that which of the following represents a biochemical variation that may exist between clients of different cultures? Risk for osteoporosis Skin cancer Motor development Drug metabolism

Drug metabolism

The nurse is beginning the examination of the skin of a 25-year-old teacher. She previously visited the office for evaluation of fatigue, weight gain, and hair loss. The previous clinician had a strong suspicion that the client has hypothyroidism. What is the expected moisture and texture of the skin of a client with hypothyroidism? Moist and smooth Moist and rough Dry and smooth Dry and rough

Dry and rough

A hospitalized client continues to exhibit residual effects of a stroke. Which symptom is the priority concern? Facial weakness Right ptosis Dysphagia Weak gait

Dysphagia

Teenagers doing community service following arrest for driving under the influence and are working at the rehabilitation hospital with clients who have paraplegia. These clients have been paralyzed by drunk drivers. How would the nurses who care for these clients best use the time spent with these teenagers? Educating them about not drinking and driving Teaching them how to turn these clients every 2 hours Fulfilling the court requirements Keeping the shelves restocked

Educating them about not drinking and driving

During assessment of the oral cavity, the nurse examines the salivary glands. Which area of the mouth should the nurse assess to inspect for the Wharton's ducts? Posterior aspect of the tongue bilaterally Right side of the frenulum at the base of the gums Buccal mucosa across from the second upper molars Either side of the frenulum on the floor of the mouth

Either side of the frenulum on the floor of the mouth

A nurse is preparing to perform auscultation on a client. Which guideline is most important for the nurse to keep in mind while performing this technique? Eliminate distracting noises from the environment. Use good lighting, preferably sunlight. Look and observe before touching the client. Compare appearance of symmetric body parts.

Eliminate distracting noises from the environment.

The nurse is discharging an adult client who received 18 staples for a head laceration received while mountain biking. What can the nurse focus on while doing discharge teaching? Encourage the use of safety equipment Encourage proper nutrition to promote healing Encourage the client to take a safety course Teach proper posture, bending, and lifting

Encourage the use of safety equipment

A client diagnosed with a peritonsillar abscess exhibits 4+ tonsils and is not able to eat or drink. What is the nurse's priority concern for this client? Ensure a patent airway Correct client's dehydration Obtain a throat culture Begin antibiotics immediately

Ensure a patent airway

A nurse is educating a client about the function of the parts of the auditory system. Which is the function of the eustachian tube? Transmits vibration to the fluid filled inner ear at the oval window. Separates the external from the middle ear. Sends sensory information to the cerebellum and midbrain. Equalizes the pressure in the middle ear with atmospheric pressure.

Equalizes the pressure in the middle ear with atmospheric pressure.

A nurse recognizes that the belief that one's worldview is the only acceptable truth and that one's beliefs, values, and sanctioned behaviors are superior to all others is called what? Ethnocentrism Stereotyping Egocentrism Ethnicity

Ethnocentrism

Revising the plan as needed occurs in what part of the nursing process? Assessment Diagnosis Planning Evaluation

Evaluation

The nurse notes a large keloid on the pierced ear of an adolescent. The client asks what caused this finding. Which of the following would the nurse incorporate into the response as the most likely cause? Continuous trauma Excessive collagen formation Decreased subcutaneous tissue Inadequate circulation

Excessive collagen formation

When performing the cover test, a nurse notices that the client's left eye turns outward. How should the nurse document this finding in the client's record? Exotropia Esotropia Strabismus Presbyopia

Exotropia

A client has just been admitted to the postsurgical unit from postanesthetic recovery, and the nurse is in the introductory phase of the client interview. Which of the following activities should the nurse perform first? Collaborate with the client to identify problems. Explain the purpose of the interview. Determine the client's vital signs. Obtain family health history data.

Explain the purpose of the interview.

An avid swimmer presents with ear pain. Her history includes pain and drainage from the left ear. On examination, she has pain when the ear, including the tragus, is manipulated. The canal is narrowed and erythematous with some white debris in the canal. The rest of the examination is normal. What diagnosis would be most appropriate for this client? Otitis media External otitis Perforation of the tympanum Cholesteatoma

External otitis

The nurse is working on a pediatric unit caring for a 4-year-old who is recovering from the surgical repair of the pelvis. When assessing the client's pain, what is the most appropriate pain assessment tool for the nurse to use? Face, Legs, Activity, Cry, Consolability Scale Visual Analog Scale FACES Pain Scale Numeric Pain Intensity Scale

FACES Pain Scale

A nurse is using a quick reference to guide the spiritual assessment. A published acronym related to the assessment of spirituality is which of the following? FACES PIE FICA PRN

FICA

A nurse assesses a cognitively impaired adult client who grimaces and points to the right knee following a motor vehicle accident. Which pain scale would be most appropriate for the nurse to use to assess the client's pain? Verbal Descriptor Scale Numeric Rating Scale Faces Pain Scale Visual Analog Scale

Faces Pain Scale

A client has been found to have abnormal vision. What would be the nurse's next step? Facilitate a referral to an ophthalmologist. Refer the client to social services to get money for glasses. Refer the client to a community-based support group. Encourage the client to adopt a healthier diet

Facilitate a referral to an ophthalmologist.

When assessing pulses, the nurse would use which part of the hand for palpation? Finger pads Ulnar surface Palmar surface Dorsal surface

Finger pads

The nursing student asks the nurse what would be an example of visceral pain. What would be the correct response by the nurse? Gallbladder pain Burn pain Cardiac pain Arthritic pain

Gallbladder pain

A client on a medical-surgical unit reports pain of 10 on a scale of 0 to 10 and wants more pain medication. The nurse does not think the pain is as bad as the client says. The physician left orders for prn morphine for breakthrough pain. What is the priority nursing action? Give the prn morphine Hold the medication and wait 30 minutes Call the physician to check the order Document the client's pain rating on a scale of 0 to 10

Give the prn morphine

A client shares that a first-degree relative has an eye problem, but they not sure what the diagnosis is. What major eye problem will the nurse suggest screening the client for? Retinoblastoma Strabismus Retinitis pigmentosa Glaucoma

Glaucoma

What is the most common type of hyperthyroidism? Graves' disease Cushing's syndrome Moon face Thyroid cancer

Graves' disease

A six-month old male infant is brought to the emergency department by his parents for inconsolable crying and pulling at his right ear. When assessing this infant the nurse is aware that the tympanic membrane should be what color in a healthy ear? Yellowish-white Red Gray Bluish-white

Gray

The nurse is preparing to perform a physical examination of a client who is an Orthodox Jew. Which of the following accommodations should the nurse be prepared to make for this client, based on his religious beliefs? Allow the client to pray before the examination Let the client remained fully dressed for the examination Have a nurse who is the same sex as the client examine him Avoid asking any questions regarding the client's lifestyle

Have a nurse who is the same sex as the client examine him

A nurse begins the eye examination on a client who presents to the health care clinic for a routine examination. What is the correct action by the nurse to perform the test for near visual acuity? Have the client hold the Jaeger card 14 inches from the face and read with one eye at a time Sit the client in front of the examiner, extend one arm, and slowly move one finger upward Tell the client to remove glasses, if present, and read the Snellen card using both eyes Place the client 20 feet from the Snellen chart and record the smallest line the client can read

Have the client hold the Jaeger card 14 inches from the face and read with one eye at a time

A client complains of pain in several areas of the body. How should the nurse assess this client's pain? Ask the client to rate the area with the highest pain level. Mark each site on the client's body with a marker. If pain does not radiate, there is no need to rate that area. Have the client rate each location separately.

Have the client rate each location separately.

The nurse is documenting an objective assessment of the client's ears. Which of the following would be the most appropriate documentation? Hearing intact bilaterally on whisper test Hearing intact on right and left with Rinne test No decrease in hearing evident on Weber test Client states experiencing no decrease in hearing

Hearing intact bilaterally on whisper test

A nurse performs an admission assessment on a client admitted with chest pain. The nurse knows that using the bell of the stethoscope is appropriate to auscultate for which type of sounds? Bowel Normal heart Breath Heart murmur

Heart murmur

Which finding should a nurse recognize as normal when assessing the ears of an elderly client? Decrease in cerumen production Shortened earlobes High-tone frequency loss Bulging tympanic membrane

High-tone frequency loss

A nurse inspects a client's nails and notes the angle between the nail base and the skin is greater than 180 degrees. What additional data should the nurse collect from this client? Onset of iron deficiency anemia History of cigarette smoking Environmental exposure to chemicals Treatment for fungal infections in the past

History of cigarette smoking

You are caring for a client in the outpatient clinic with suspicion of cancer due to recent weight losses for unidentifiable reasons. The client a 25-year history of smoking. You perform an assessment and ask the client about symptoms related to laryngeal cancer. What is an early symptom associated with laryngeal cancer? Alopecia Dyspnea Hoarseness Dysphagia

Hoarseness

You are caring for a client in the outpatient clinic with suspicion of cancer due to recent weight losses for unidentifiable reasons. The client a 25-year history of smoking. You perform an assessment and ask the client about symptoms related to laryngeal cancer. What is an early symptom associated with laryngeal cancer? Hoarseness Alopecia Dysphagia Dyspnea

Hoarseness

A nurse is assessing the effect of a client's chronic back pain on his affective dimension. Which question should the nurse ask for this assessment? What medical conditions do you have? Where is the pain located? What is the highest level of education you've completed? How does the pain influence your overall mood?

How does the pain influence your overall mood?

When providing client teaching about the ears, what should the nurse be sure to include? How the client cleans the ears Basic anatomy and physiology of the ears How to use cotton-tipped applicators safely Potential infection from self-cleaning of ears

How the client cleans the ears

What structure is found midline in the tracheal area just beneath the mandible? Cricoid cartilage Hyoid bone Thyroid cartilage Adam's apple

Hyoid bone

A client presents to the emergency department after being hit in the head with a baseball bat during a game. The nurse should assess for which condition? Hyphema Blepharitis Chalazion Iris nevus

Hyphema

A client presents at the clinic for a routine check-up. The nurse notes that she is dressed in warm clothing even though the temperature outside is 73°F (22.8°C). The nurse also notes that the client has gained 10 pounds (4.5 kg) since her last visit 9 months ago. What might the nurse suspect? Effects of age-related changes Brain tumor Hyperthyroidism Hypothyroidism

Hypothyroidism

A nurse has completed assessment of a client with Alzheimer's disease and documentation of the information obtained from the client and now needs to analyze the data collected. Which nursing actions should be included in this phase of the nursing process? Select all that apply. Identification of collaborative problems Assessment of the outcome of the care plan Identification of the need for referrals Formulation of nursing diagnosis(es) Development of a nursing care plan

Identification of collaborative problems Identification of the need for referrals Formulation of nursing diagnosis(es)

A nurse assesses the spirituality of a client who is terminally ill with pancreatic cancer. Which of the following is the best rationale for this action, from the nurse's perspective? Searching for meaning and purpose Finding common practices shared with a group Identifying possible coping mechanisms Defining the meaning of the afterlife

Identifying possible coping mechanisms

While the nurse is assessing a client for an unrelated health concern, the client experiences a sudden, severe headache with no known cause. He also complains of dizziness and trouble seeing out of one eye. What associated condition should the nurse suspect in this client? Diabetes Brain tumor Impending stroke Hyperthyroidism

Impending stroke

A client is diagnosed with an obstruction of the canal of Schlemm affecting the left eye. What assessment data concerning the left eye noted in the client's medical record supports this diagnosis? Increased intraocular pressure Sluggish pupillary reaction Displaced optic nerve Opaque lens

Increased intraocular pressure

A client tells the nurse about a raised lesion on the client's leg. What is the nurse's first nursing action? Inspect the area Ask further questions Document the statement Move on to next body system

Inspect the area

A nurse is completing an assessment that will involve gathering subjective and objective data. Which of the following assessment techniques will best allow the nurse to collect objective data? Inspection Therapeutic communication Interviewing Active listening

Inspection

A nurse is beginning the physical examination of an elderly man with chronic obstructive pulmonary disease. In which order should the nurse implement the four physical assessment techniques with this client? Auscultation, percussion, palpation, inspection Percussion, palpation, inspection, auscultation Palpation, inspection, auscultation, percussion Inspection, palpation, percussion, auscultation

Inspection, palpation, percussion, auscultation

A nurse assesses the distant vision acuity of a client using the Snellen chart. Which action should the nurse implement to perform the test with accuracy? Position the client 12 feet away from the Snellen chart Instruct the client to read without reading glasses Instruct the client to lean forward and read the chart Ask the client to cover one eye with the hand

Instruct the client to read without reading glasses

A nurse asks a client to rate his pain on a scale of 0 to 10, with 0 being no pain and 10 being worst pain. What characteristic of pain is the nurse assessing? Duration Location Chronology Intensity

Intensity

When entering a client's room, the nurse washes the hands and asks the client for his name. What is the nurse's next best action? Introduce self and explain what will be done. Auscultate heart, lungs, and breath sounds. Prepare medications in front of the client. Take the client's vital signs and record them.

Introduce self and explain what will be done.

Which of the following is inconsistent with acute otitis media? The infection usually lasts less than 6 weeks. It is a relatively uncommon childhood infection. A purulent exudates is usually present in the middle ear. Conductive hearing loss may occur.

It is a relatively uncommon childhood infection.

A client on hospice care tells his nurse that he is not afraid of death. He explains that this is because he believes he has lived a very good life and therefore his next life should be even better than this one, as all of his life actions will be carried into the next. The nurse recognizes this belief as which of the following? Ramadan Holy Trinity Kosher Karma

Karma

A client comes to the clinic and reports nosebleeds. What area of the nose is the bleeding most likely coming from? Sinuses Kiesselbach plexus Thompson plexus Wharton ducts

Kiesselbach plexus

A nurse is providing client teaching to the parents of a preschooler who experiences chronic epistaxis. What would the nurse identify as the area where most nosebleeds originate? Stensen's duct Wharton's ducts Kiesselbach's plexus Rosenmuller's fossa

Kiesselbach's plexus

A nurse is teaching a group of 5th grade children about characteristics of the skin. Which of the following should she mention? Select all that apply. Largest organ of the body Involved in digestion of food Protects against damage to the body from sunlight Circulates blood throughout the body Helps make vitamin D in the body Aids in maintaining body temperature

Largest organ of the body Protects against damage to the body from sunlight Helps make vitamin D in the body Aids in maintaining body temperature

A nurse assesses a client with regard to nutritional habits, use of substances, education, and work and stress levels. The nurse recognizes this as what type of information? History of present health concern Personal health history Family health history Lifestyle and health practices profile

Lifestyle and health practices profile

Which is the priority for the nurse conducting a physical examination of a client with generalized muscle weakness? Limit position changes as much as possible Hand-washing throughout the exam Using alcohol swabs to clean the stethoscope Draping body areas that are not being assessed.

Limit position changes as much as possible

Which would the nurse recognize as an example of visceral pain? Select all that apply. Liver pain Gallbladder pain Pancreatic pain Burn pain Muscular pain

Liver pain Gallbladder pain Pancreatic pain

Which of the following are elements of a pain assessment? (Select all that apply.) Location Duration Intensity Aggravating/alleviating factors Quality

Location Duration Intensity Aggravating/alleviating factors Quality

The nurse understands the importance of performing an accurate pain assessment. In addition to having the client rate the pain on a pain scale, other things to assess are the following: (Check all that apply.) Location and duration Quality and description Diet and allergies Alleviating and aggravating factors Urine output and pulse oximetry value

Location and duration Quality and description Alleviating and aggravating factors

A client has returned from outpatient surgery where a biopsy revealed a nodule on the laryngopharynx. The client asks where this is located. What is the best response by the nurse to the client about the location of laryngopharynx? Part of the pharynx extending from the uvula to the epiglottis Connects the nasopharynx to the middle ear Section of the pharynx from the nares to the uvula Lowest portion of the pharynx

Lowest portion of the pharynx

During the physical examination of your client you auscultate the sound of the client's breathing. What area of the client are you assessing? Abdomen Neck Lungs Back

Lungs

After completing a spiritual assessment, a nurse determines that the client has signs and symptoms requiring medical diagnosis and treatment. What should the nurse do? Return to the client to further discuss spirituality. Make a referral to a primary care provider. Treat the medical condition. Ignore the medical problem.

Make a referral to a primary care provider.

A nurse is examining a client who is complaining of sinus pressure in his face and congestion. The nurse discovers tenderness on palpation of the sinuses and a large amount of exudate. Over which sinuses should the nurse expect to feel crepitus in this client? Sphenoidal Frontal Maxillary Ethmoidal

Maxillary

When examining the eye with an ophthalmoscope, where would the nurse look to visualize the optic disc? Medially toward the nose Laterally toward the ear Upward toward the forehead Downward toward the chin

Medially toward the nose

A client reports right-sided temporal headache accompanied by nausea and vomiting. A nurse recognizes that which condition is likely to produce these symptoms? Bell's palsy Tension headache Temporal arteritis Migraine headache

Migraine headache

The nurse is assessing a client and notices that when being asked questions, the client does not make eye contact. The nurse should include what in the plan of care? History of abuse Minimal eye contact due to culture Fear of authority Uncomfortable answering questions.

Minimal eye contact due to culture

The nurse assess the client for common upper respiratory symptoms which may include what? Select all that apply. Nasal congestion Fever Pain when breathing Hoarseness Pharyngitis

Nasal congestion Hoarseness Pharyngitis

Which area should the nurse inspect for facial symmetry when performing a head and neck assessment? Nasolabial folds Temporomandibular joint Preauricular nodes Earlobe placement

Nasolabial folds

A client recovering from a stroke complains of pain. The nurse suspects this client is most likely experiencing which type of pain? Nociceptive Neuropathic Somatic Idiopathic

Neuropathic

When teaching a class of school-age children about hygiene, the nurse should include which information about the ears? Ears should be cleaned with cotton applicators once a day at bedtime. Never put anything smaller than your elbow in your ears. Only allow your parents clean out your ears with cotton applicators. Producing less earwax can lead to a hearing loss.

Never put anything smaller than your elbow in your ears.

The nurse palpates a client's auricles and notes an enlarged lymph node on one ear. No redness is observed, and the client denies pain or tenderness. What is the nurse's best action? Document the finding as an isolated benign node. Inform the client of the need for ear drops. Refer the client to an audiologist for an audiogram. Notify the healthcare provider about the finding.

Notify the healthcare provider about the finding.

A client complains of a unilateral headache near the scalp line and double vision. The nurse palpates the space above the cheekbone near the scalp line on the affected side, and the client complains of tenderness on palpation. What is the nurse's next action? Notify the healthcare provider immediately. Administer intravenous pain medication. Palpate the carotid pulses bilaterally at the same time. Prepare the client for a temporal artery biopsy.

Notify the healthcare provider immediately.

A client presents to the clinic reporting sudden visual loss in the left eye. What is the nurse's priority action? Assess cranial nerve function. Notify the healthcare provider immediately. Ask the client if protective eyewear was worn. Perform the Allen test and report the findings urgently.

Notify the healthcare provider immediately.

An adolescent client who suffered a crushing leg injury due to an all terrain vehicle (ATV) rollover, complains of painful tingling in the affected leg and inability to bend the knee. The client's leg is swollen. What is the nurse's priority action? Administer the prescribed non-steroid anti-inflammatory medication. Elevate the client's leg on several pillows and apply ice. Raise the head of the bed to High Fowler's position. Notify the healthcare provider immediately.

Notify the healthcare provider immediately.

An older adult client had hip replacement surgery 2 days ago. The nurse enters the client's room and encourages the client to use the incentive spirometer ten times every hour. What is this action an example of? Nursing intervention Nursing goal Nursing evaluation Nursing assessment

Nursing intervention

A nurse has completed the assessment of a client's direct pupillary response and is now assessing consensual response. This aspect of assessment should include which action? Observing the eye's reaction when a light is shone into the opposite eye Shining a light into one eye while covering the other eye with an opaque card Have the client state when they see the nurse's finger enter their peripheral vision field. Comparing the difference between the client's dilated pupil and a constricted pupil

Observing the eye's reaction when a light is shone into the opposite eye

What is the common channel for the respiratory and digestive systems? Nares Sinuses Oropharynx Frenulum

Oropharynx

A nurse is caring for a client who is ambulating for the first time after surgery. Upon standing, the client complains of dizziness and faintness. The client's blood pressure is 90/50. What is the name for this condition? Orthostatic hypotension Orthostatic hypertension Ambulatory bradycardia Ambulatory tachycardia

Orthostatic hypotension

Which of the following describes a condition characterized by abnormal spongy bone formation around the stapes? Otosclerosis Middle ear infection Chronic otitis media Otitis externa

Otosclerosis

The nursing instructor is discussing the administration of nasal spray with the nursing students. What information is most important to include in this discussion? Overuse of nasal spray may cause rebound congestion. Nasal spray can be shared between family members only. Administer the nasal spray in a prone position. Finish the bottle of nasal spray to clear the infection effectively.

Overuse of nasal spray may cause rebound congestion.

Which of the following principles should the nurse integrate into the pain assessment and pain management of pediatric clients? Pain assessment may require multiple methods in order to ensure accurate pain data. The developing neurological system children transmits less pain than in older clients. Pharmacologic pain relief should be used only as an intervention of last resort. A numeric scale should be used to assess pain if the child is older than 5 years of age.

Pain assessment may require multiple methods in order to ensure accurate pain data.

You are a pediatric nurse caring for a child who has been brought to the clinic with otitis externa. What assessment finding is characteristic of otitis externa? Tophi on the pinna and ear lobe Dark yellow cerumen in the external auditory canal Pain on manipulation of the auricle

Pain on manipulation of the auricle

A client has been brought to the emergency unit of a health care facility following an automobile accident. Which finding about the lips supports the diagnosis of anemia and shock? Reddish Pallor Swelling Cyanotic

Pallor

During a physical examination of a client, the nurse assesses the size of the liver. Which of the following techniques should the nurse use for this assessment? Inspection Palpation Percussion Auscultation

Palpation

Which of the following health problems necessitates the use of an ophthalmoscope during assessment? Papilledema Cataracts Subconjunctival hemorrhage Tonic pupil

Papilledema

As a novice nurse caring for a client from a different culture, what may the nurse find confusing or upsetting? Patient's level of motivation Patient's nonverbal communication Patient's family members Patient's ethnicity

Patient's nonverbal communication

On a health history, a client reports no visual disturbances, last eye exam being 2 years ago, and not wearing glasses. The nurse notices that the client squints when signing the consent for treatment form and holds the paper close to the face. What should the nurse do next? Document the findings in the client's record Perform both the distant and near visual acuity tests Test the pupils for direct and consensual reaction to light Obtain a referral to the ophthalmologist for a complete eye exam

Perform both the distant and near visual acuity tests

A client reports feeling short of breath. Which area of the body should the nurse inspect for the presence of cyanosis? Perioral Palms Facial Chest

Perioral

Which characteristic of the gums should a nurse expect to assess in a client who is healthy? Enlarged, reddened A grey-white line Pink, moist, firm Red, bleeding

Pink, moist, firm

Which precaution should a nurse take to ensure the safety of a client when performing the Romberg test? Instruct the client to hold on to a chair Offer assistance by holding the client's arm Place arms around the client without touching Tell the client to keep the eyes open & focused ahead

Place arms around the client without touching

The nurse is assessing the hearing of an older adult. Which type of hearing problem might the nurse expect to find in the older adult? Presbycusis Tinnitus Conductive hearing loss Sensorineural hearing loss

Presbycusis

Which terms refers to the progressive hearing loss associated with aging? Presbycusis Exostoses Otalgia Sensorineural hearing loss

Presbycusis

A nurse notices a middle-aged client in the waiting room pick up a magazine to read while she waits to be seen. She opens the magazine and then extends her arms to move it further from her eyes. Which condition does the nurse most suspect in this client? Exotropia Esotropia Strabismus Presbyopia

Presbyopia

The nurse observes a middle-aged colleague fully extending her arm to read the label on a vial of medication. Which of the following age-related changes is the nurse likely to have observed? Presbyopia Cataract formation Loss of convergence Macular degeneration

Presbyopia

When assessing a client's respirations, what is most important to include in the documentation? Numerical pain rating Position of the client Assessment of pedal pulses Presence of dyspnea

Presence of dyspnea

Which technique should the nurse use to examine the sinuses of a client with a sinus infection? Inspect the frontal and maxillary sinuses with an otoscope Insert a penlight into the oral cavity & angle it toward the roof of the mouth Press up on the brow on each side of the nose to palpate the frontal sinus. Indirectly percuss over the cheekbones for dullness or pain

Press up on the brow on each side of the nose to palpate the frontal sinus.

A client reports severe pain in the posterior region of the neck and difficulty turning the head to the right. What additional information should the nurse collect? Previous injuries to the head and neck Difficulty with swallowing Changes in sleeping habits Stiffness in the right shoulder

Previous injuries to the head and neck

Parents bring a child to the clinic and report a "rash" on her knee. On assessment, the nurse practitioner notes the area to be a reddish-pink lesion covered with silvery scales. What would the nurse practitioner chart? Seborrhea Contact dermatitis Eczema Psoriasis

Psoriasis

A nurse assesses a client for past history of nail problems. The nurse should ask questions about which of these conditions? Psoriasis, fungal infections, trauma Vitiligo, hirsutism, vitamin deficiency Eczema, melanoma, herpes zoster Alopecia, dermatitis, chemotherapy

Psoriasis, fungal infections, trauma

When testing the near reaction, an expected finding includes which of the following? Pupillary dilation on near gaze; dilation on distant gaze Pupillary dilation on near gaze; constriction on distant gaze Pupillary constriction on near gaze; dilation on distant gaze Pupillary constriction on near gaze; constriction on distant gaze

Pupillary constriction on near gaze; dilation on distant gaze

A young adult client has just had X-rays and computed tomography scanning of the head and neck following a mountain bicycling accident. All results are negative. What should the nurse assess for next? Range of motion of the neck Headache Shortness of breath Range of motion of the arms and shoulders

Range of motion of the neck

While discussing care needs a client states the desire to be positive about the upcoming medical treatments and expect them to help cure the disease. Which nursing diagnosis should the nurse identify for this client? Hopelessness Spiritual distress Risk for spiritual distress Readiness for enhanced hope

Readiness for enhanced hope

A nurse is assessing the mouth of an older client. Which of the following findings is common among older adults? Bifid uvula Brown spots on the chewing surface of teeth Receding and ischemic gums Enlarged palatine tonsils

Receding and ischemic gums

Which assessment of the tongue should a nurse recognize as abnormal? Pale pink and moist Fissured, topographical pattern Red with loss of papillae Ventral surface with visible veins

Red with loss of papillae

Which characteristic of the gums should a nurse expect to assess in a client who has scurvy? Red, bleeding Pink, moist, firm Enlarged, reddened A grey-white line

Red, bleeding

Which characteristic feature of the tympanic membrane should a nurse anticipate finding in a client with acute otitis media? Pearly, translucent, with no bulging Yellowish, bulging, with fluid bubbles Gray, translucent, with no retraction Red, bulging, with an absent light reflex

Red, bulging, with an absent light reflex

A nurse is examining the nose of a client diagnosed with an upper respiratory tract infection. Which characteristics of the nasal mucosa should the nurse expect to find during assessment of a client with an upper respiratory tract infection? Bluish gray, swollen, with watery exudate Red, swollen, with purulent discharge Pale pink, swollen, with watery exudate Dark pink, moist, & free of discharge

Red, swollen, with purulent discharge

A client who is semiconscious is brought to the emergency department of a health care facility after being rescued from a fire. Which finding of the lips supports the diagnosis of carbon monoxide poisoning? Cyanotic Reddish Swelling Pallor

Reddish

The nurse notes otitis media with effusion in the left ear of a 3-year-old child. Which assessment data is consistent with otitis media with effusion? Redness and bulging of the eardrum Clear discharge in the ear canal Bloody discharge in the ear canal Dense white patches on the tympanic membrane

Redness and bulging of the eardrum

What would be the expected tone elicited by percussion of a normal lung? Resonance Hyper-resonance Tympany Dullness

Resonance

On examining a client, the nurse detects a foul odor to the breath. The nurse recognizes this finding as a characteristic of what disease process? Small bowel obstruction End-stage liver disease Respiratory infection Diabetic ketoacidosis

Respiratory infection

A nurse is caring for a client that was in a motor vehicle accident and suffered damage to the external ear. The client asks the nurse how hearing will be affected on a long-term basis. Based on this information, what is the best description the nurse can tell the client about the function of the external ear? Responsible for protecting internal structures of the ear from foreign substances. Responsible for the transmission of sound. Responsible for the transmitting sound waves. Responsible for providing information about body position to the brain.

Responsible for protecting internal structures of the ear from foreign substances.

What part of the eye receives and transmits visual stimuli to the brain for processing? Retina Optic disc Posterior chamber Vitreous chamber

Retina

What auscultation sound would the nurse expect when listening to heart sounds? Swooshing Rhythmic lub-dub Rustling Gurgly

Rhythmic lub-dub

A client reports recent bilateral hearing loss. Which of the following tests should the nurse use to determine if the hearing loss is sensorineural or conductive? Rinne Weber whisper Romberg

Rinne

Which of the following tests use a tuning fork between two positions to assess hearing? Whisper Watch tick Rinne Weber's

Rinne

A client presents to the health care clinic and reports pain in the eyes when working on the computer for long periods of time. The client states that he almost ran into a parked car yesterday because he misjudged the distance from the bumper of his own car. He works for a computer software company and has noticed he is experiencing difficulty reading the manuals that accompany the software he installs for companies. What nursing diagnosis can the nurse confirm based on this data? Ineffective Individual Coping Disturbed Self Concept Self-Care Deficit Risk for Injury

Risk for Injury

A client with advanced presbycusis admits to the nurse that he was nearly involved in a car accident because he could not hear the siren of an ambulance that was crossing an intersection through which he was driving. The client says that he lives alone and has no one else to drive him. Which of the following diagnoses can the nurse make at this time? Risk for Injury related to hearing impairment Impaired Social Interaction related to hearing loss Risk for Loneliness related to hearing loss Readiness for Enhanced Communication related to expressed desire for a hearing aid

Risk for Injury related to hearing impairment

A client asks how long the hospitalization will be since family and friends will not be able to visit because of the distance to travel. Which nursing diagnosis should be identified for this client's concern? Anxiety Hopelessness RC: Depression Risk for social isolation

Risk for social isolation

A client admitted to the health care facility is diagnosed with vertigo. Which test is appropriate for the nurse to perform to assess for equilibrium in the client? Weber Rinne Romberg Whisper

Romberg

A client with a family history of melanoma wants to have specific body moles assessed. In order to perform this assessment effectively, the nurse should have access to what equipment? Select all that apply. Warm water Ruler Gloves Magnifying glass Natural lighting

Ruler Gloves Magnifying glass Natural lighting

A client is assigned a visual acuity of 20/100 in her left eye. Which of the following is true? She obtains a 20% correct score at 100 feet. She can accurately name 20% of the letters at 20 feet. She can see at 20 feet what a normal person could see at 100 feet. She can see at 100 feet what a normal person could see at 20 feet.

She can see at 20 feet what a normal person could see at 100 feet.

A nurse should assist a client to assume what position to best assess the mouth, nose, and sinuses? Semi-recumbent position, with the chin lifted Sitting with the head erect and at the eye level of the nurse Tilting the head backwards, with the neck flexed Prone, with arms relaxed at the sides

Sitting with the head erect and at the eye level of the nurse

The nurse is presenting an educational event for gardeners. When discussing the ears, what would be an important topic to cover? Skin cancer prevention Otalgia Tinnitus Sound control

Skin cancer prevention

A nurse is preparing to perform a physical examination of an obese client who is beginning a diet and exercise program. The physician would like to establish a baseline percent body fat measurement for the client so that the client's progress in reducing body fat can be tracked over time. Which piece of equipment should the nurse anticipate needing for this purpose? Platform scale with height attachment Metric ruler Sphygmomanometer Skinfold calipers

Skinfold calipers

On examination of a client, the nurse detects a fecal odor to the breath. The nurse recognizes this finding as characteristic of what disease process? End-stage liver disease Respiratory infection Diabetic ketoacidosis Small bowel obstruction

Small bowel obstruction

The nurse is caring for a 63-year-old client who can neither read nor speak English. What would be the appropriate chart to use to assess this client's vision? Allen Snellen E Ishihara PERRLA

Snellen E

A client recovering from abdominal surgery is complaining of pain. The nurse realizes that the client is most likely experiencing which type of pain? Psychogenic Idiopathic Neuropathic Somatic

Somatic

An elderly farmer has sustained severe injuries after a serious accident involving a combine harvester. At the hospital, he tells the nurse that he thinks the pain he is feeling now is "payback" for living a "mean, selfish life." The nurse recognizes that this response by the man indicates which dimension of pain? Cognitive dimension Sociocultural dimension Affective dimension Spiritual dimension

Spiritual dimension

The client states "What does it matter? I have no purpose in this life." This client is questioning what? Religion Spirituality Cultural beliefs Cultural practices

Spirituality

A 12-year-old presents to the clinic with his father for evaluation of a painful lump in the left eye. It started this morning. The client denies any trauma or injury. There is no visual disturbance. Upon physical examination, there is a red raised area at the margin of the eyelid that is tender to palpation; no tearing occurs with palpation of the lesion. Based on this description, what is the most likely diagnosis? Dacryocystitis Chalazion Stye Xanthelasma

Stye

Which glands are responsible for mouth drainage? Select all that apply. Sebaceous Lacrimal Submandibular Sublingual Parotid

Submandibular Sublingual Parotid

does the first sound heard through the stethoscope represent? Systolic pressure Diastolic pressure Auscultatory gap Pulse pressure

Systolic pressure

Upon entering an adult client's room to begin a shift assessment, the nurse should call the rapid response team based on which assessment finding? Systolic pressure 180 mm Hg. Apical pulse 70 beats/minute. Respirations 12 breaths/minute. Oxygen saturation 95% on room air.

Systolic pressure 180 mm Hg.

A nurse is preparing to assess a client's vital signs. In which order should the nurse assess them? Blood pressure, temperature, pulse, and respirations Respirations, blood pressure, pulse, and temperature Temperature, pulse, respirations, and blood pressure Pulse, temperature, respirations, and blood pressure

Temperature, pulse, respirations, and blood pressure

The nurse is explaining the difference between acute pain and chronic pain to the client. Which should the nurse include in the explanation? The cause of acute pain can be identified. The duration of chronic pain is short. Chronic pain is caused by damage to nerves. Acute pain lasts longer than 3 to 6 months.

The cause of acute pain can be identified.

An adult client is having his skin assessed. The client tells the nurse he has been a heavy smoker for the last 40 years. The client has clubbing of the fingernails. What does this finding tell the nurse? The client has chronic hypoxia The client has melanoma The client has COPD The client has asthma

The client has chronic hypoxia

A nurse is admitting a client to the hospital. When reviewing the client's medical record, the nurse notes that this client had abnormal findings during the Weber test. What would the nurse know this means? The client has unilateral hearing loss The client has loss of high-frequency sounds The client has loss of low-frequency sounds The client has bilateral hearing loss

The client has unilateral hearing loss

Which action by the nurse is consistent with Weber's test? The nurse activates the tuning fork and places it on the midline of the parietal bone in line with both ears. The nurse strikes the tuning fork and places it on the client's mastoid process to measure bone conduction. The nurse uses a bulb insufflator attached to an otoscope to observe movement of the tympanic membrane. The nurse shields their mouth and whispers a simple sentence approximately 18 inches from the client's ear.

The nurse activates the tuning fork and places it on the midline of the parietal bone in line with both ears.

Which action by the nurse is consistent with the Rinne test? The nurse strikes the tuning fork and places it on the client's mastoid process to measure bone conduction. The nurse activates the tuning fork and places it on the midline of the parietal bone in line with both ears. The nurse uses a bulb insufflator attached to an otoscope to observe movement of the tympanic membrane. The nurse shields their mouth and whispers a simple sentence approximately 18 inches from the client's ear.

The nurse strikes the tuning fork and places it on the client's mastoid process to measure bone conduction.

A client reports after a back massage that his lower back pain has decreased from 8 to 3 on the pain scale. What opioid neuromodulator may be responsible for this increased level of comfort? The release of endorphins The release of insulin The release of melatonin The release of dopamine

The release of endorphins

When examining the head, the nurse remembers that the anatomic regions of the cranium take their names from which of the following sources? Noted anatomists The underlying bones Their anatomical positions The underlying vascular network

The underlying bones

A client has an abnormal consensual pupillary reaction to light. A nurse understands that what reaction occurs in the client's eyes? Pupils dilate in response to a light shone in the eyes. Eyes do not converge to focus on a shining light. There is no reaction in the opposite pupil to light. Light reflection appears at different spots on both eyes.

There is no reaction in the opposite pupil to light.

The nurse is assessing a client complaining of swelling in the neck. While palpating the neck, the nurse finds a 2-cm lump that is fixed and hard. Why does this finding require emergency investigation? This could be a sign of cancer This could be a sign of pneumothorax This could be a sign of an embolus This could be a sign of a parotid stone

This could be a sign of cancer

A 4-year-old child presents to the health care clinic with circular lesions. Which of the following conditions should the nurse most suspect in this client, based on the configuration of the lesions? Multiple nevi Tinea versicolor Herpes simplex Tinea corporis

Tinea corporis

During a pharmacology class the students are told that some drugs need to be closely monitored. What aspect should the nurse closely monitor for in clients who have been administered salicylates, loop diuretics, quinidine, quinine, or aminoglycosides? Signs of hypotension Reduced urinary output Tinnitus and sensorineural hearing loss Impaired facial movement

Tinnitus and sensorineural hearing loss

A client injures his thumb by accidently slamming the car door shut on it. He arrives at the emergency department in intense pain. Which of the following processes is associated with the transduction process of this pain? Inflammation leading to conduction of an impulse to the spinal cord Emotional response and rational interpretation and response Tissue injury leading to inflammation Changes or inhibitions to the pain message relay in the spinal cord

Tissue injury leading to inflammation

While interviewing a client, the nurse asks, "What happens when you have low blood glucose?" This type of response to the client is used for what purpose? To summarize the conversation To restate what the client has said To promote objectivity To clarify

To clarify

What is the primary function of the health care team? To work together to obtain maximum coverage To decide the best overall care To guide the client's care throughout times of crisis To develop an individual focus for each member

To decide the best overall care

A nursing instructor is discussing the purposes of health assessment. What is one purpose of health assessment? To establish a database against which subsequent assessments can be measured To establish rapport with the client and family To gather information for specialists to whom the client might be referred To quantify the degree of pain a client may be experiencing

To establish a database against which subsequent assessments can be measured

Before calling a client back to an examination room, the nurse quickly observes the client in the waiting room from head to toe. Which of the following is the best rationale for this action? To check the client for skin lesions the client may not be aware of To overhear the client's conversation with a family member To see the client before the client assumes a social face or behavior To determine whether you recognize the client from a previous visit

To see the client before the client assumes a social face or behavior

Which of the following assessment findings of the mouth, nose, and throat of an older adult client would the nurse attribute to the aging process? Tongue fissures Peritonsillar abscess Deviated septum Candidiasis

Tongue fissures

When assessing a client the nurse notes that the tonsils are touching the uvula. How would the nurse document the tonsils? Tonsils are T2 Tonsils are T4 Tonsils are T3 Tonsils are T1

Tonsils are T3

What is the most important focus area for the integumentary system? UV radiation exposure Chemical exposure Moles with defined borders smaller than 6 mm Washing the face and hands

UV radiation exposure

How should the nurse place the ear of an adult when using the otoscope? Up and back Down and back Up and forward Down and forward

Up and back

How can a nurse accurately assess the distant visual acuity of a client who is non-English speaking? Move an object through the six cardinal positions of gaze Use a Snellen E chart to perform the examination Have the client read from a Jaeger reading card Perform the confrontation test

Use a Snellen E chart to perform the examination

A nurse is collecting subjective data from a client who is reporting pain. Which of the following actions should the nurse take to determine the severity of the pain? Ask the client how long the pain lasts. Determine when the pain first started. Use a pain scale to measure the pain. Question the client about the character of the pain.

Use a pain scale to measure the pain.

A teenager is brought to the clinic for a sports physical examination. The client states plans to play goalie on the community soccer team. What is the most important teaching opportunity presented for this client? Use of safety equipment Prevention of knee injuries Prevention of head injuries Use of correct foot gear

Use of safety equipment

The nurse is providing care to a newly admitted client with a long history of chronic obstructive pulmonary disease (COPD). According to the client's chart, the client has been taking several inhalers to manage their respiratory condition. The nurse enters the room with the prescribed inhalers to administer them. What action should the nurse take next? Leave the inhalers with the client to self-administer. Validate that the client understands how to use the inhalers. Ask the client if they need any assistance with the inhalers. Provide privacy for the client to administer the inhalers.

Validate that the client understands how to use the inhalers.

When visualizing the structures of the nose, the nurse recalls that air travels from the anterior nares to the trachea through the: Ala nasi, turbinates, and nasopharynx Turbinates, ethmoid sinuses, and nasal passages Vestibule, nasal passages, and nasopharynx Ala nasi, vestibule, and ethmoid sinuses

Vestibule, nasal passages, and nasopharynx

A nurse is caring for a client with dull ache in her abdomen. On the way to the health care facility, the client vomits and shows symptoms of pallor. What kind of pain is the client experiencing? Visceral pain Cutaneous pain Somatic pain Neuropathic pain

Visceral pain

Mark is a 20-year-old college student who has been experiencing increasingly sharp pain in the right, lower quadrant of his abdomen over the last 12 hours. A visit to the emergency department and subsequent diagnostic testing have resulted in a diagnosis of appendicitis. What category of pain is Mark most likely experiencing? Visceral pain Referred pain Cutaneous pain Somatic pain

Visceral pain

You are teaching a physiology class for pre-nursing students. A student asks what the purpose of the upper airway is in regard to the lower airway. What would be your best answer? Clean the inspired air Warm the expired air Warm the inspired air Clean the expired air

Warm the inspired air

The nurse is assessing a client's respiratory rate. Which of the following should the nurse do to ensure accuracy of this assessment? Watch chest movement before removing the stethoscope after counting the apical beat Ask the client to breathe normally Observe the client's chest movement before calling the client back to the examination room Perform the assessment at the beginning, middle, and end of the examination and average the results

Watch chest movement before removing the stethoscope after counting the apical beat

The submandibular glands open under the tongue through openings called Wharton ducts. parasinal ducts. Stensen ducts. lacrimonasal ducts.

Wharton ducts.

The nurse learns that a client routinely engages in spiritual practices. Which activities should the nurse document as being spiritual in nature? Select all that apply. Yoga Prayer Knitting Baking bread Attending church

Yoga Prayer Attending church

The nurse should make it a priority to assess which client for papilledema? a 45-year-old suspected of experiencing a subarachnoid hemorrhage an 80-year-old diagnosed with chronic open-angle glaucoma a 12-year-old demonstrating a deviated left eye a 56-year-old reporting double vision

a 45-year-old suspected of experiencing a subarachnoid hemorrhage

The nurse is assessing a dark-skinned client who has been transported to the emergency room by ambulance. When the nurse observes that the client's skin appears pale, with blue-tinged lips and oral mucosa, the nurse should document the presence of a great degree of cyanosis. a mild degree of cyanosis. lupus erythematosus. hyperthyroidism.

a great degree of cyanosis.

A nurse performs a comprehensive assessment on a client. The nurse observes the following findings: enlarged hands, feet, and facial features (nose, ears). Which of the following disorders do these findings indicate? Parkinson disease Cushing syndrome scleroderma acromegaly

acromegaly

After examining the client's tympanic membranes, the nurse documents "Right tympanic membrane, red and bulging with no light reflex." The nurse recognizes that these are signs of acute otitis media. serous otitis media. skull trauma. trauma from infection.

acute otitis media.

The nurse is planning to assess an adult client's thyroid gland. The nurse should plan to ask the client to raise the chin. approach the client posteriorly. turn the client's neck slightly backward. place the fingers above the cricoid cartilage.

approach the client posteriorly.

The nurse is preparing to perform a head and neck assessment of an adult client who has immigrated to the United States from Cambodia. The nurse should first explain to the client why the assessment is necessary. ask the client if touching the head is permissible. determine whether the client desires a family member present. examine the lymph nodes of the neck before examining the head.

ask the client if touching the head is permissible.

During an interview, a client of Hispanic descent reports receiving most health care from a folk healer. The nurse anticipates that the client participates in which of the following preventative health measures? using masks to ward off evil spirits using talismans and amulets avoiding the evil eye balancing yin/yang

avoiding the evil eye

When assessing the client for pain, the nurse should doubt the client when he or she describes the pain. assess for underlying causes of pain, then believe the client. believe the client when he or she claims to be in pain. assess for the presence of physiologic indicators (such as diaphoresis, tachycardia, etc.), then believe the client.

believe the client when he or she claims to be in pain.

While assessing an adult client's feet for fungal disease using a Wood light, the nurse documents the presence of a fungus when the fluorescence is blue. red. yellow. purple.

blue.

The conjunctiva of the eye is divided into the palpebral portion and the canthus portion. intraocular portion. nasolacrimal portion. bulbar portion.

bulbar portion.

The nurse is conducting a physical examination of a client who is in the lying position. Place in order the areas the nurse will assess when completing this examination. a. Shins and ankles b. Groin, hips, and knees c. Breasts d. Chest and thorax e. Cardiovascular

c, d, e, b, a

A client's blood pressure is affected by cardiac intake, elasticity of the arteries, blood flow, blood cells, and blood thickness. cardiac intake, elasticity of the veins, blood flow, blood cells, and blood thickness. cardiac output, distensibility of the veins, blood volume, blood velocity and viscosity. cardiac output, distensibility of the arteries, blood volume, blood velocity and viscosity.

cardiac output, distensibility of the arteries, blood volume, blood velocity and viscosity.

The middle layer of the eye is known as the choroid layer. scleral layer. retinal layer. optic layer.

choroid layer.

The chambers of the eye contain aqueous humor, which helps to maintain intraocular pressure and transmit light rays. maintain the retinal vessels. change refractory of the lens. cleanse the cornea and the lens.

cleanse the cornea and the lens.

A client rates the current pain level as being a 5 on the Numeric Rating Scale. How should the nurse document this pain assessment? client experiencing a moderate amount of pain. client experiencing mild pain. client rated pain level as being a 5 using the rating scale. client stated "pain level not that bad."

client rated pain level as being a 5 using the rating scale.

The transmission of sound waves through the external ear and the middle ear is known as perceptive hearing. conductive hearing. external hearing. connective hearing.

conductive hearing.

A client is concerned because the sclera of the right eye has been pink in color for several days and tearing. What should the nurse suspect is occurring with this client? hyphema anisocoria conjunctivitis exophthalmos

conjunctivitis

The nurse notes that the pupil of a client's left eye constricts when a light is shined into the right eye. How should the nurse document this finding? direct light response present in left eye pupils equal and react to accommodation consensual light response present in left eye consensual light response present in right eye

consensual light response present in left eye

A nurse is performing a focused visual assessment on a client. The nurse assesses the pupillary response with a pen light. Both of the client's pupils immediately constrict when the light is shone into the right pupil. How should the nurse document this finding? consensual reflexes observed negative for nystagmus equal accommodation positive corneal reflex

consensual reflexes observed

The nurse learns that a client is unable to sleep because of high anxiety. On which category of health patterns should the nurse focus? sleep-rest activity-exercise coping-stress-tolerance self-perception/self-concept

coping-stress-tolerance

A nurse should assess clients for signs of spiritual distress, which include which of the following? (Select all that apply.) crying wishing to die singing anger

crying singing anger

A male nurse is caring for an elderly woman who has become withdrawn and somewhat confused since the nurse has come on duty. When the nurse goes into the room to bathe the client, she refuses to allow it. The best explanation for her actions would be which of the following? confusion depression fear in strange surroundings cultural differences

cultural differences

An elderly client is seen by the nurse in the neighborhood clinic. The nurse observes that the client is dressed in several layers of clothing, although the temperature is warm outside. The nurse suspects that the client's cold intolerance is a result of: decreased body metabolism. neurologic deficits. recent surgery. pancreatic disease.

decreased body metabolism.

A client returns to the unit after a thyroidectomy. On entering the client's room, the nurse observes the client having difficulty breathing due to swelling in the neck. What type of assessment should the nurse perform at this time? ongoing or partial comprehensive emergency focused

emergency

The nurse observes an inward turning of the lower lid in a 77-year-old client. The nurse documents entropion ectropion ptosis exophthalmos

entropion

While assessing the eye of an adult client, the nurse observes an inward turning of the client's left eye. The nurse should document the client's esotropia. strabismus. phoria. exotropia.

esotropia.

Total parenteral nutrition (TPN) has been prescribed for a client. After several hours of infusion, the nurse checks the client's glucose and it is elevated, requiring insulin. The nurse administers the insulin as prescribed. What step in the nursing process should the nurse take next? assessment diagnosis planning evaluation

evaluation

An assessment of a client who already has a complete recorded database in the system and returns to the health care agency with a specific health concern is referred to as a(n) ongoing or partial assessment. focused or problem-oriented assessment. emergency assessment. initial comprehensive assessment.

focused or problem-oriented assessment.

A nurse performs a focused assessment on a new client. The nurse observes pustules and erythema around the client's hair follicles. The nurse recognizes these are signs and symptoms of which of the following disorders? alopecia folliculitis ringworm tinea capitis

folliculitis

The result of a nursing assessment is the: prescription of treatment. documentation of the need for a referral. client's physiologic status. formulation of nursing diagnoses.

formulation of nursing diagnoses.

The nurse is planning to assess a client's near vision. Which technique should be used? shine a light on the bridge of the nose have the client read newspaper print held 14 inches from the eyes ask the client to move the eyes in the direction of a moving finger have the client stand 20 feet from a wall chart and read the letters after covering one eye

have the client read newspaper print held 14 inches from the eyes

The nurses assesses the thyroid gland of a client with recent weight loss. On auscultation, a low, soft, rushing sound is heard over the lateral lobes. Which condition is most likely? hyperthyroidism thyroid cyst Hashimoto thyroiditis benign tumor

hyperthyroidism

A nurse is performing a comprehensive assessment on a client. The nurse observes pale, cyanotic nails with a 180-degree angle with spongy sensation and clubbing of the distal ends of the fingers. The nurse identifies these signs and symptoms as indications of which of the following conditions? iron deficiency anemia fungal infection psoriasis hypoxia

hypoxia

An adult client visits the clinic and tells the nurse that he has been experiencing double vision for the past few days. The nurse refers the client to a physician for evaluation of possible glaucoma. increased intracranial pressure. hypertension. ophthalmic migraine.

increased intracranial pressure.

The most commonly used method of percussion is: direct percussion. mild percussion. indirect percussion. blunt percussion.

indirect percussion.

Light palpation is most appropriate to assess the: appendix bladder inflamed areas of skin liver

inflamed areas of skin

A nurse is assessing a client who is reporting a sore throat. The nurse should further assess the client for which of the following common underlying causes of throat pain? Select all that apply. influenza measles common cold cold sore gingivitis

influenza measles common cold

During a comprehensive assessment, the primary technique used by the nurse throughout the examination is: palpation. percussion. auscultation. inspection.

inspection.

An adult client visits the clinic and tells the nurse that he has had excessive tearing in his left eye. The nurse should assess the client's eye for viral infection. double vision. allergic reactions. lacrimal obstruction.

lacrimal obstruction.

When the client reports a problem associated with the drainage of tears from the left eye, the nurse would focus the eye assessment on which eye structure? lacrimal gland lacrimal puncta lacrimal sac nasolacrimal duct

lacrimal puncta

An older client cannot recall the date of a surgical procedure but the adult daughter interjects with the exact date because it occurred a week before her wedding. How should the nurse document this information? adult daughter controlling the interview unable to recall exact date of last surgery last surgery date validated by adult daughter confused regarding dates of surgical procedures

last surgery date validated by adult daughter

An older client asks why vision is not as sharp as it used to be when the eyes are focused forward. What should the nurse realize this client is describing? cataracts glaucoma detached retina macular degeneration

macular degeneration

A client visits the clinic for a routine physical examination. The nurse prepares to assess the client's skin. The nurse asks the client if there is a family history of skin cancer and explains to the client that there is a genetic component with skin cancer, especially basal cell carcinoma. actinic keratoses. squamous cell carcinoma. malignant melanoma.

malignant melanoma.

The tarsal plates of the upper eyelid contain meibomian glands. sebaceous glands. tear ducts. ocular muscles.

meibomian glands.

A female client visits the clinic and tells the nurse that she frequently experiences severe recurring headaches that sometimes last for several days and are accompanied by nausea and vomiting. The nurse determines that the type of headache the client is describing is a migraine headache. cluster headache. tension headache. tumor-related headache.

migraine headache.

The current blood pressure measurement on a 24-hour uncomplicated postoperative client while standing at the bedside is 105/65. The last two readings were 130/75 and 125/70 while resting in bed. The nurse should be alert for signs of: orthostatic hypotension. supine hypotension. hypertensive crisis. postural hypertension.

orthostatic hypotension.

During the focused assessment, the client experiences pain when the nurse manipulates (wiggles) the pinna of the ear. Based on these findings, the nurse suspects the client may be experiencing which of the following disorders? otalgia tinnitus otitis media otitis externa

otitis externa

The nurse is completing a physical examination of a client who reports ear pain. In order to determine if the tympanic membrane is still intact, which instrument is required? otoscope sphygmomanometer stethoscope ophthalmoscope

otoscope

A client has a 10-year history of being treated for hypertension. Where should the nurse document this information? health patterns review of systems health maintenance past medical history

past medical history

A nurse is caring for a female client. The client tells the nurse that in their culture, the husband makes all the decisions. The nurse understands that this is common in some societies that believe in which of the following family structures? paternalistic generational Western-style autonomous

paternalistic

Transmission of sound waves in the inner ear is known as conductive hearing. tympanic hearing. neuromotor hearing. perceptive hearing.

perceptive hearing.

The nurse is preparing to inspect the nose of an adult client with an otoscope. The nurse plans to direct the otoscope tip quickly back and down the nostril. position the handle of the otoscope straight and up. position the handle of the otoscope to one side. tip the client's head as far back as possible.

position the handle of the otoscope to one side.

The nurse is planning to assess a client's lymph nodes. Which set of nodes should the nurse assess first? submental preauricular supraclavicular superficial cervical

preauricular

The nurse notes that a client's nails are greater than a 160-degree angle. What should the nurse assess as a priority for this client? heart sounds bowel sounds pulse oximetry body temperature

pulse oximetry

The nurse assesses the client's vital signs as follows: respirations 20 breaths/minute, tympanic temperature 100.9°F, pulse 88 beats/minute, and blood pressure 104/64 mm Hg. The nurse should: record the vital signs. instruct the client to drink more fluids. refer the client to a primary care provider. administer Tylenol.

record the vital signs.

The nurse assesses an adult client's head and neck. While examining the carotid arteries, the nurse assesses each artery individually to prevent: reduction of the blood supply to the brain. rapid rise in the client's pulse rate. premature ventricular heart sound. decreased pulse pressure.

reduction of the blood supply to the brain.

A client has tested 20/40 on the distant visual acuity test using a Snellen chart. The nurse should document the results in the client's record. ask the client to read a handheld vision chart. ask the client to return in 2 weeks for another examination. refer the client to an optometrist.

refer the client to an optometrist.

A client reports pressure in the chest but describes pain in the jaw and right arm. Based on these findings, the nurse determines that the client may be experiencing which of the following types of pain? phantom radicular referred inflammatory

referred

Photoreceptors of the eye are located in the eye's ciliary body. lens. retina. pupil.

retina

The nurse is preparing to assess the lymph nodes of an adult client. The nurse should instruct the client to lie in a supine position. lie in a side-lying position. stand upright in front of the nurse. sit in an upright position.

sit in an upright position.

A client reports pain in the knee. The knee is warm, swollen, and red and the client describes the pain as aching and gnawing. The nurse determines the client is experiencing which of the following types of nociceptive pain? neuropathic somatic referred phantom

somatic

A client has been admitted to a medical floor with a diagnosis of pneumonia. Upon admission, the client is talkative and friendly to the nurse. When the nurse approaches the bed to listen to the client's lung sounds, the client becomes quiet and starts to pull away. The nurse understands that this reaction is most likely related to the client's perception of what? silence autonomy time space

space

Matters of the human soul are referred to as what? culture ethnicity values and beliefs spirituality

spirituality

The nurse assesses a bed-bound older adult client in the client's home. While assessing the client's buttocks, the nurse observes that an area of the skin is broken. The wound is shallow and dry, and there is no bruising. The nurse should document the client's pressure ulcer as stage I. stage II. stage III. stage IV.

stage II.

A nurse is providing end-of-life care for a client who practices the Hinduism faith. The nurse anticipates the family will request to perform which of the following at the time of the client's death? an autopsy immediate cremation straightening of the client's arms positioning the client to face east

straightening of the client's arms

When performing a cultural assessment, an important point to remember would be: that definitions of family differ to use first names of those you are speaking to that alternative therapies are reasons for seeking care that the cultural/ethnic background is evident in the client's appearance

that definitions of family differ

The nurse documents information about a client's activity-exercise health pattern. Which information did the nurse most likely document? gained 15 lbs. over the last 6 months experiences panic attacks several times a week unable to go to the gym since having back surgery misses seeing friends who used to go for walks together

unable to go to the gym since having back surgery

The nurse is interviewing a client in the clinic for the first time. The client appears to have a very limited vocabulary. The nurse should plan to: use very basic lay terminology. have a family member present during the interview. use standard medical terminology. show the client pictures of different symptoms, such as the "faces pain chart."

use very basic lay terminology.

A nurse is conducting a cultural assessment. Which of the following should the nurse include in the assessment? Select all that apply. values affiliations physical health history of mental illness communication

values affiliations communication

Short, pale, and fine hair that is present over much of the body is termed vellus. dermal. lanugo. terminal.

vellus.

Sensory receptors in the ear that help to maintain both static and dynamic equilibrium are located in the semicircular canals and the vestibule. tympanic membrane. cone of light. eustachian tube.

vestibule.

A client is admitted with right lower abdominal pain with rebound tenderness. The nurse suspects appendicitis and documents this type of pain as which of the following? phantom somatic visceral referred

visceral

The nurse is assessing an elderly postsurgical client in the home. To begin the physical examination, the nurse should first assess the client's: height and weight. ability to swallow. vital signs. gait.

vital signs.

The rich blood supply of the nose serves to propel debris to the throat. help propel moist air to the body. filter large particles from the air. warm the inspired air.

warm the inspired air.

A nurse performs a hearing test on an elderly client. Which result should the nurse recognize as an indication that presbycusis is present? An inability to hear: whispered sounds the calling bell the tuning fork the sound of "th"

whispered sounds

During an interview between a nurse and a client, the nurse and the client collaborate to identify problems and goals. This occurs during the phase of the interview termed: introductory. ongoing. working. closure.

working.

A clinic nurse has reviewed a new client's available health record and will now begin taking the client's health history. Which of the following questions should the nurse ask first when obtaining the health history? "Do you have adequate health insurance coverage?" "Are you generally fairly healthy?" "What is your major health concern at this time?" "Did you bring all your medications with you?"

"Did you bring all your medications with you?"

A nurse is assessing the cognitive function of a 13-year-old boy who is in the hospital following a head injury sustained while playing football. The boy acts annoyed with the assessment questions and asks how often he will have to answer them. The nurse should respond with which of the following? "Fortunately, assessment only needs to be done at the beginning of your stay." "I'll just need to evaluate you once more, at the end of your stay." "Typically, assessment occurs once at the beginning of your stay, once in the middle, and once at the end." "I'm sorry, but assessment is ongoing and continuous."

"I'm sorry, but assessment is ongoing and continuous."

A nurse has a regular client who is an immigrant from China and who follows the traditional medical system of that culture. To improve the ability to understand and work with this client, the nurse researches this client's cultural view of health. Which of the following would the nurse most likely discover? A focus on maintaining balance between yin and yang Respect for nature and use of masks and sand paintings Prayers to God and saints for spiritual reparations for sins Use of herbs, roots, talismans, and amulets

A focus on maintaining balance between yin and yang

A client with cancer is talking to the primary nurse about the diagnosis and states, "I feel like it is God's will that I have this cancer. If I get out of the hospital, I want to take nature walks like I used to before I got so sick. I feel so close to God when I do this." The nurse understands that this client is demonstrating which of the following? Traumatization over this diagnosis Acceptance of the diagnosis Viewing the cancer as punishment for past lifestyle Signs of depression

Acceptance of the diagnosis

A nurse reviews a client's chart. The nurse learns that the client has been eating only 25% of their meals and has lost 2 lb. (.9 kg.) since admission. The client practices Orthodox Judaism. What action(s) would a culturally competent nurse take? Select all that apply. Ask the client about food preferences. Allow family members to bring food to the client. Allow the client to arrange times they would like to eat their meals. Ask the kitchen to bring the client more of the same food they have been receiving. Tell the client, "You need to eat in order to get better."

Ask the client about food preferences. Allow family members to bring food to the client. Allow the client to arrange times they would like to eat their meals.

The nurse is caring for a client exhibiting slurred speech after suffering from a cerebrovascular accident. The nurse is unable to completely understand the client. What is the nurse's best action? Change the subject to put the client at ease. Turn the television on for distraction. Ask the client to repeat the statement or question. Refer all questions to the client's family member in room.

Ask the client to repeat the statement or question.

Which assessment finding should the nurse document as objective data? Biographical information Body functions Lifestyle practices Personal relationships

Body functions

Which of the following statements is true about biologic variation? Both genetics and environment produce biologic variation. Cultural practices produce biologic variation. Race is based on physical variations. Drug metabolism differences are not culture based.

Both genetics and environment produce biologic variation.

A client who practices the Christian faith has terminal stage 4 cancer. The client tells the nurse, "God will save me." What is the best action by the nurse? Inform the client that stage 4 cancer is rarely curable. Educate the client regarding stage 4 cancer and survival outcomes. Request a psychiatric consult to determine if the client has cognitive deficits. Continue to provide care and support the client's religious beliefs.

Continue to provide care and support the client's religious beliefs.

The nurse observes that a client who is Jewish and follows an Orthodox way of life. The client is not eating the food that the dietary department is sending. What would be the best lunch to bring this client? Cottage cheese, peaches, and crackers with tea. Roast beef sandwich, mashed potatoes, and a cup of ice cream. Sliced pork, stuffing, and green peas. Crab cakes, french fries, and a roll.

Cottage cheese, peaches, and crackers with tea.

A new graduate nurse from a small community college is going to work in an urban setting. She embraces the thought of working in a more diverse setting and wants to learn about different cultures. What behavior is this nurse exhibiting? Cultural desire Cultural knowledge Cultural skill Conscious incompetence

Cultural desire

The nurse working in the emergency room has been assigned the following clients. Which client requires an ongoing assessment? A.) a client admitted with acute atrial fibrillation who has a heart rate of 150 bpm and irregular heart rhythm B.) a client admitted with a leg fracture who is reporting sudden shortness of breath and a rash C.) a newly admitted client who was involved in a motor vehicle incident with a head injury and reports a headache of 3 on a scale of 1-10 D.) a client admitted 2 days ago with exacerbation of chronic obstructive pulmonary disease with an oxygen saturation of 90% on 2L nasal cannula who reports ease of breathing

D.) a client admitted 2 days ago with exacerbation of chronic obstructive pulmonary disease with an oxygen saturation of 90% on 2L nasal cannula who reports ease of breathing

One technique of therapeutic communication is silence. What does silence allow the client to do? Learn to trust the nurse Change topics if he or she wants Communicate concerns nonverbally Decide how much information to disclose

Decide how much information to disclose

A nurse completes an initial assessment and discusses findings with the client. What is the next best action of the nurse? Perform a review of systems. Develop a plan of care with the client. Validate the client's biographical data. Discuss lifestyle and health practices with the client.

Develop a plan of care with the client.

A nurse on an oncology unit enters a client's room to auscultate bowel sounds. What should the nurse do before auscultating? Disinfect the stethoscope before touching the client Disinfect the stethoscope after touching the client Make sure the stethoscope is placed directly on the client's skin so that there is complete contact with the skin surface Put on a personal protection gown

Disinfect the stethoscope before touching the client

The nurse would use what part of the hand when assessing temperature during palpation? Dorsal surface Finger pads Ulnar surface Palmar surface

Dorsal surface

Staff are talking to the hospital educator and ask about "a government project that is meant to improve the health of people in the United States." The educator bases her response on the knowledge of: Healthy People 2030 the nursing process the Department of Health and Human Services the three levels of preventative care

Healthy People 2030

x A nursing instructor is teaching about spirituality, major world religions, and common beliefs. The instructor realizes that a student understands the differences based on which of the following statements? Hindus believe that illness is the result of past and current life actions. Buddhists believe that Allah is in control of the beginning and end of life. Jews believe that prayer and meditation are used for cleansing and healing. Christians believe that restrictions related to work on holy days are removed to save a life.

Hindus believe that illness is the result of past and current life actions.

A nursing instructor is talking about nonverbal communication with the nursing class. The instructor explains that facial expressions should be what? Happy Inquisitive Relaxed Detached

Relaxed

A client asks the nurse if there are church services in the hospital because the client attends mass every Sunday. The nurse realizes that this client is demonstrating: Religion Culture Spirituality Recreation

Religion

During an interview, the client begins to talk about the frequency of being abused by a spouse. What can the nurse do at this time to acknowledge the sensitivity of the information the client is providing? Write down the information as the client is speaking. Key the information into the electronic medical record as the client is speaking. Avoid maintaining eye contact while the client is discussing spouse abuse. Stop documenting in order to maintain eye contact with the client.

Stop documenting in order to maintain eye contact with the client.

The nurse is educating a client from another country about the medications they will be taking. The client continually interrupts the nurse during the conversation. When the nurse considers the many ways cultural differences can affect communication, how should the nurse interpret the client's interruptions? This client is impolite and does not know good manners. The client believes that what they have to say is more important than what the nurse is saying. The client is deeply engaged in the conversation. The client believes that they know more about the medications than the nurse does.

The client is deeply engaged in the conversation.

The nurse is setting an outcome with the client experiencing spiritual distress. Which goal would be appropriate? The client will express meaning and purpose in life. The client will attend a community event every week. The client will express feelings of connectedness with others. The client will initiate interaction with others.

The client will express meaning and purpose in life.

A Muslim teenager is in the hospital for surgical repair of a severe fracture in the leg during the month of Ramadan, which the client says he would like to observe during his stay. Which of the following interventions should the nurse be prepared for in caring for this client? Withhold all food and drink between sunrise and sunset Serve meals as normal, but do not include pork Serve only kosher meats at all three meals Do not mix dairy and meats together on the client's tray

Withhold all food and drink between sunrise and sunset

The nurse is caring for a client in the health care provider's office. In reviewing the client's chart, the nurse recognizes the need for providing the client with additional education related to COVID-19 when noting which of the following about the client? Works in the service industry Lives in a high-income household Has a history of low blood pressure Eats a diet low in carbohydrates

Works in the service industry

The nurse is beginning a health history interview with an adult client who expresses anger at the nurse. The best approach for dealing with an angry client is for the nurse to: allow the client to verbalize his or her feelings. offer reasons why the client should not feel angry. provide structure during the interview. refer the client to a different health care provider.

allow the client to verbalize his or her feelings.

After performing a comprehensive assessment on a client, the nurse notes the following. Which part of the nursing process is the nurse performing? Nursing Notes: ● Client reports pain in bilateral lower extremities when walking short distances, relieved with rest. ● Pulses are weak, barely palpable in bilateral lower extremities. ● Bilateral feet are cool to touch ● Total cholesterol > 200. ● Client smokes two packs of cigarettes daily for past 20 years. documentation of subjective assessment findings development of priority nursing diagnosis analysis of assessment findings implementation of interventions

analysis of assessment findings

A nursing instructor is teaching how to perform a spiritual assessment and informs students that it is best not to ask uncomfortable questions but instead observe for which of the following characteristics? (Select all that apply.) client's concept of God client's sexual preference client's religious practices client's food preferences

client's concept of God client's religious practices

When the nurse tells the client that she will return in 1 hour to administer an intravenous antibiotic, the nurse is showing that she values time in which of the following orientations? future past present none of the above

future

The nurse has interviewed a Hispanic client with limited English skills for the first time. The nurse observes that the client is reluctant to reveal personal information and believes in a hot-cold syndrome of disease causation. The nurse should: indicate acceptance of the client's cultural differences. request a family member to interpret for the client. use slang terms to identify certain body parts. remain in a standing position during the interview.

indicate acceptance of the client's cultural differences.

The nurse tells a newly admitted client that she is going to do a health assessment to help in planning care and educational needs during the client's hospital stay. Before the physical examination, the nurse should first: take a complete health history formulate a plan of care make a list of appropriate nursing diagnoses collect all home medications brought to the hospital

take a complete health history


Kaugnay na mga set ng pag-aaral

GW- Ch 9- CompTIA Security+ (Exam SYO-501)

View Set

Chapter 5: How to Form a Business

View Set

CH2 Traits, Motives, and Characteristics of Leaders

View Set

Adult Health and Illness 1 Final Exam Study Questions

View Set

ATI - The Hematologic System Test

View Set